Talk:2018 AMC 12B Problems/Problem 13

Revision as of 23:59, 24 October 2021 by MRENTHUSIASM (talk | contribs) (Put the discussion of the Shoelace Theorem on this page since I don't really want to complicate things up. I'd like to keep the solution page clean. Let me know if you disgree with this action.)
(diff) ← Older revision | Latest revision (diff) | Newer revision → (diff)

Remark (Coordinate Geometry)

Alternatively, use the Shoelace Theorem once you have found the coordinates in terms of $x$ and $y$. The $x$s and $y$s will conveniently cancel and the answer is (still) $\boxed{\textbf{C}}$.

~Fasolinka